Can you help me solve the one with mrs Jones I want to know if I’m right

Can You Help Me Solve The One With Mrs Jones I Want To Know If Im Right

Answers

Answer 1

We know that

• 11 students own a cat.

,

• 12 students own a dog.

,

• 6 students own both a cat and a dog.

,

• 3 students own neither.

First, let's draw a Venn diagram to visualize the problem.

First, we have to fund the total number of students inside the sets Cat and Dog. We need to subtract the number 6 once, otherwise, we'll count it twice.

[tex]11+12-6=17[/tex]

Then, we include the students that own neither.

[tex]17+3=20_{}[/tex]Therefore, the total number of students is 20.
Can You Help Me Solve The One With Mrs Jones I Want To Know If Im Right

Related Questions

Write the fraction as equivalent fraction with the given denominator

Answers

Okay, here we have this:

Considering the provided fraction, we are going to rewrite it as equivalent fraction with the given denominator, so we obtain the following:

Then we will solve the following proportion to find the missing value:

[tex]\frac{3}{4}=\frac{x}{12}[/tex]

Solving for x:

[tex]\begin{gathered} x=\frac{3}{4}\cdot12 \\ x=\frac{36}{4} \\ x=9 \end{gathered}[/tex]

Finally we obtain the following fractions:

[tex]\frac{3}{4}=\frac{9}{12}[/tex]

Answer: 9/12

Step-by-step explanation:

I answered a problem for my prep guide, I just need to know if I’m correct or not. And I would like it to be answered as well just to make sure that I did everything correctly

Answers

Notice that,

[tex]f(x)=3^{x-1}-6=3^x\cdot3^{-1}-6=\frac{3^x}{3}-6[/tex]

And there are no restrictions for the values that x can take. The domain is the whole set of real numbers.

Now, we need to check for the limits when x->+/- infinite, as follows:

[tex]\begin{gathered} \lim _{x\to\infty}3^x=\infty \\ \lim _{x\to-\infty}3^x=\lim _{x\to\infty}\frac{1}{3^x}=0 \end{gathered}[/tex]

Then, the range of 3^x is (0, infinite).

Finally, we can get the range of function f(x):

[tex]\lim _{x\to\infty}f(x)=\frac{1}{3}(\lim _{x\to\infty}3^x)-6=\frac{1}{3}\infty-6=\infty[/tex][tex]\lim _{x\to-\infty}f(x)=\frac{1}{3}(\lim _{x\to-\infty}3^x)-6=\frac{1}{3}\cdot0-6=-6[/tex]

Then,

[tex]\begin{gathered} The\text{ range of }f(x)\text{ is} \\ Range=(-6,\infty) \end{gathered}[/tex]

Since f is parallel to line g, use the diagram to the right right to answer the following question

Answers

Step 1

[tex]\begin{gathered} m\angle2=m\angle6=117^o(\text{ corresponding angles are equal)} \\ m\angle6=m\angle7=117^o(vertically\text{ opposite angles are equal)} \\ \end{gathered}[/tex]

Step 2

[tex]undefined[/tex]

Please help will mark Brainly

Answers

Answer:x=7

Step-by-step explanation:

Determine if the expression -4c5 + c3 is a polynomial or not. If it is a polynomial, state the type and degree of the polynomial. .

Answers

It is a polynomial. 5th degree (incomplete) polynomial. Binomial.

1) Considering the expression:

[tex]-4c^5+c^3[/tex]

2) And the Polynomial definition as:

[tex]P(x)=a_nx^n+a^{}_{n-1}x^{n-1}+.\ldots+a_0[/tex]

We can state that this is an incomplete polynomial.

About the degree, it is a 5th-degree polynomial given by its highest exponent.

Binomial since it has two terms.

3) Hence the answer is an incomplete polynomial, 5th degree.

May I please get help with Solve for x: −3<−10(x+15)≤7

Answers

Given the compound inequality;

[tex]-3<-10(x+15)\le7[/tex]

We would begin by simplifying the parenthesis as follows;

[tex]\begin{gathered} -3<-10(x+15) \\ \text{AND} \\ -10(x+15)\le7 \end{gathered}[/tex]

We shall now solve each part one after the other;

[tex]\begin{gathered} -3<-10(x+15) \\ -3<-10x-150 \\ \text{Collect all like terms and we'll have;} \\ -3+150<-10x \\ 147<-10x \\ \text{Divide both sides by -10} \\ \frac{-147}{10}>x \end{gathered}[/tex]

We can switch sides, and in that case the inequality sign would also "flip" over, as shown below;

[tex]\begin{gathered} \frac{-147}{10}>x \\ \text{Now becomes;} \\ x<\frac{-147}{10} \end{gathered}[/tex]

For the other part of the compound inequality;

[tex]\begin{gathered} -10(x+15)\le7 \\ -10x-150\le7 \\ \text{Collect all like terms and we'll have;} \\ -10x\le7+150 \\ -10x\le157 \\ \text{Divide both sides by -10} \\ \frac{-10x}{-10}\le\frac{157}{-10} \\ x\ge-\frac{157}{10} \end{gathered}[/tex]

Therefore, the values are;

[tex]\begin{gathered} x<-\frac{147}{10} \\ \text{And } \\ x\ge-\frac{157}{10} \\ \text{Hence;} \\ -\frac{157}{10}\le x<-\frac{147}{10} \end{gathered}[/tex]

Written in interval notation, this now becomes;

[tex]\lbrack-\frac{157}{10},-\frac{147}{10})[/tex]

I’m trying to find out where the second point can be marked

Answers

ANSWER

First point = (0, 3)

Second point = (1, -1)

Third point = (2, -5)

Graph:

EXPLANATION

To plot a graph using the slope and the y-intercept, simply apply the following rules:

1. Evaluate the function at x = 0, to determine the y-intercept which was (0,3) from the question

2. Determine the slope by finding the change in y divided by change in x. This was -4 according to the question. Which could also be written as -4/1; that is, rise divided by run

3. Now, from the value (0, 3) we got in step 1, we move down by 4 units and then to the right by 1 unit. This will lead us to the Second point of (1, -1). Also from this point, we move down by 4 units and then to the right by 1 unit to get to the Third point of (2, -5). You may decide to continue this pattern if you want more points.

4. Draw a straight line joining the 3 points together.

which expression are equivalent to[tex]( \frac{750}{512})^{ \frac{1}{3} } [/tex]

Answers

[tex](\frac{750}{512})^{\frac{1}{3}}[/tex]

Fractional exponents refer to the radicals

Option A (Correct)

[tex]\frac{\sqrt[3]{750}}{\sqrt[3]{512}}[/tex]

Option B (Incorrect)

750 is not a perfect cube

Option C (Correct)

[tex]\sqrt[3]{\frac{750}{512}}[/tex]

Option D (Incorrect)

The denominator does not have the root

Option E (Incorrect)

The numerator does not have the root

Option F (Correct)

[tex]\frac{5}{8}\sqrt[3]{6}[/tex]

(Please reference attached photo for problem.)Show your work please. Also, What is the perimeter?

Answers

Solution:

Given the shape below:

The above shape is a combination of a semicircle and a rectangle labeled as A and B respectively.

To find the perimeter of the shape:

step 1: Evaluate the perimeter of the circle.

The perimeter of the semicircle is expressed as

[tex]\begin{gathered} perimeter\text{ of semicircle=2}\pi r \\ where\text{ r is the radius} \\ \pi\Rightarrow3.14 \end{gathered}[/tex]

Thus, we have

[tex]\begin{gathered} perimeter=2\times3.14\times(\frac{10}{2}) \\ =31.4\text{ cm} \end{gathered}[/tex]

step 2: Evaluate the perimeter of the rectangle.

The perimeter of the rectangle is expressed as

[tex]\begin{gathered} perimeter=2(l+w) \\ where \\ l\Rightarrow length \\ w\Rightarrow width \end{gathered}[/tex]

In this case, we have

[tex]\begin{gathered} l=10\text{ cm} \\ w=4\text{ cm} \\ thus, \\ Perimeter\text{ = 2\lparen10+4\rparen} \\ =2(14) \\ =28\text{ cm} \end{gathered}[/tex]

step 3: Sum up the perimeters.

Thus, we have

[tex]\begin{gathered} perimeter\text{ of shape = perimeter of circle + perimeter of rectangle} \\ =31.4+28 \\ \Rightarrow perimeter\text{ of shape = 59.4 cm} \end{gathered}[/tex]

Hence, the perimeter of the shape is evaluated to be

[tex]59.4\text{ cm}[/tex]

Mr. Crow, the head groundskeeper at High Tech Middle School, mows the lawn along the side of the gym. The lawn is rectangular, and the length is 5 feet more than twice the width. The perimeter of the lawn is 250 feet.a. Define a variable and write an equation for this problem.b. Solve the equation that you wrote in part (a) and find the dimensions of the lawn. c. Use the dimensions you calculated in part (a) to find the area of the lawn.

Answers

Given:

Length of the rectangular lawn is 5 feet more than the width.

Perimeter of the lawn is 250 feet.

The objective is,

a) To define the variables and write an equation.

b) To solve the equation in part (a) and find the dimensions of the lawn.

c) To find the area of the lawn.

a)

Consider the width of the lawn as w feet.

It is give that the length of the lawn is 5 feet more than the width of the feet.

Then, length of the law can be represented as, l = w+5.

Since, the perimeter is given as 250 feet, the equation can be represented as,

[tex]\begin{gathered} P=2(l+w) \\ P=2(w+5+w) \\ P=2(2w+5)_{} \\ P=4w+10 \end{gathered}[/tex]

Hence, the required equation is P = 4w+10.

b)

Now, the dimensions of the lawn can be calculated by substituting the value of perimeter in the equation.

[tex]\begin{gathered} 250=4w+10 \\ 4w=250-10 \\ 4w=240 \\ w=\frac{240}{4} \\ w=60\text{ f}eet \end{gathered}[/tex]

Since, the length of the lawnis 5 feet more than the width of the lawn.

[tex]\begin{gathered} l=w+5 \\ l=60+5 \\ l=65\text{ f}eet \end{gathered}[/tex]

Hence, the width of the lawn if 60 feet and length of the lawn is 65 feet.

c)

Area of rectangluar lawn can be calculated as,

[tex]\begin{gathered} A=l\times w \\ A=65\times60 \\ A=3900ft^2 \end{gathered}[/tex]

Hence, the area of the lawn is 3900 square feet.

Notation scientific ad and subtract2.4 *10^5 + 0.5*10^5 =

Answers

We will operate as follows:

[tex]2.4\cdot10^5+0.5\cdot10^5=2.9\cdot10^5[/tex]

Dana rode her bike for 5 miles on Wednesday. On Thursday, she biked 4 1/3 times as far ason Wednesday. How many miles did Dana bike on Thursday?fraction or as a whole or mixed number.

Answers

First, let's express the mixed number as a fraction:

[tex]4\text{ }\frac{1}{3}=\frac{4\cdot3+1}{3}=\frac{13}{3}[/tex]

She rode her bike for 5 miles on wednesday and on thursday she biked 13/3 times as far as on wednesday, so:

5 miles * (13/3) =

[tex]5\times\frac{13}{3}=\frac{65}{3}\approx21.667miles[/tex]

Rosalie is training for a marathon. She jogs for 30 minutes at a rate of 5 miles per hour then she decreases her speed over a period of time and walks for 60 minutes at a rate of 3 miles per hourWhat is the range of this relation

Answers

Answer:

A. 3 ≤ y ≤ 5

Explanation:

The range is the set of values that the variable y can take. In this case, the variable y is the speed, so the range is the set of values of Rosalie's speed in her training.

Since the speed takes values from 3 miles per hour to 5 miles per hour, the range is

3 ≤ y ≤ 5

Kiera is decorating for a party. She wants balloons in 6 different locations. In each location, she will have 3 bunches of 4 balloons. How many balloons will Kiera need in all?

Answers

3 x 4 = 12 balloons in each location
6 x 12 = 70 balloons needed in total

Mrs. Everett is shopping for school supplies with her children. Rose selected 3 one-inch binders and 1 two-inch binder, which cost a total of $23. Judy selected 5 one-inch binders and 3 two-inch binders, which cost a total of $49. How much does each size of binder cost?

Answers

We define the following variables:

• x = cost of one-inch blinders,

,

• y = cost of two-inch blinders.

From the statement of the problem, we know that:

• Rose selected 3 one-inch blinders and 1 two-inch blinder, which cost a total of $23, so we have that:

[tex]3x+y=23,[/tex]

• Judy selected 5 one-inch blinders and 3 two-inch blinders, which cost a total of $49, so we have that:

[tex]5x+3y=49.[/tex]

We have the following system of equations:

[tex]\begin{gathered} 3x+y=23, \\ 5x+3y=49. \end{gathered}[/tex]

We must solve the system of equations using the elimination method, where you either add or subtract the equations to get an equation in one variable.

1) We multiply the first equation by 3, and we have:

[tex]\begin{gathered} 9x+3y=69, \\ 5x+3y=49. \end{gathered}[/tex]

2) Now, we subtract the second equation to the first equation:

[tex]\begin{gathered} (9x+3y)-(5x+3y)=69-49. \\ 4x=20, \\ x=\frac{20}{4}=5. \end{gathered}[/tex]

3) Replacing the value x = 5 in the second equation, and solving for y we get:

[tex]\begin{gathered} 5\cdot5+3y=49, \\ 25+3y=49, \\ 3y=49-25, \\ 3y=24, \\ y=\frac{24}{3}=8. \end{gathered}[/tex]

We have found that:

[tex]\begin{gathered} x=5, \\ y=8. \end{gathered}[/tex]

Answer

A one-inch binder costs $5, and a two-inch binder costs $8.

In rectangle ABCD, the diagonals intersect at E. If m angle∠AEB=  3x and m angle∠DEC= x+80, find m angle∠AEB and m angle∠EBA.

Answers

Since the angles∠ AEB and ∠DEC are vertically opposite angles, they are congruent, so we have:

[tex]\begin{gathered} 3x=x+80 \\ 2x=80 \\ x=40 \end{gathered}[/tex]

So the measure of angle ∠AEB is:

[tex]\begin{gathered} \angle\text{AEB}=3x \\ \angle\text{AEB}=3\cdot40=120\degree \end{gathered}[/tex]

The diagonals of a rectangle are congruent and intersect in their middle point, so the segment AE is congruent to the segment EB, therefore the triangle AEB is isosceles, so the angle ∠BAE is congruent to ∠EBA.

The sum of the internal angles of a triangle is 180°, so in triangle AEB we have:

[tex]\begin{gathered} \angle\text{BAE}+\angle\text{EBA}+\angle\text{AEB}=180\degree \\ \angle\text{EBA}+\angle\text{EBA}+120=180 \\ 2\angle\text{EBA}=60 \\ \angle\text{EBA}=30\degree \end{gathered}[/tex]

A private college advertise that last year their freshman students on average how do you score of 1140 on the college entrance exam. Assuming that the average refers to the mean, Which of the following claims must be true based on this information? Last year some of their freshman students had a score of exactly 1140 on the exam last year more than half of their freshman students had a score of at least 1140 on the exam last year all their freshman students have a score of at least 1140 on the exam next year at least one of their freshman students will have a score of at least 1140 on theexam last year at least one of their freshman students had a score of more than 900 on the exam or none of the above statements are true

Answers

We know that the mean score obtained by the freshman students last year was 1140.

It means that the sum of all the freshman students' scores from last year, divided by the number of freshmen students resulted in the number 1140.

It doesn't mean necessarily that one or more students had a score of exactly 1140.

Step 1

Find an example showing that some of the statements must not be true.

A way of obtaining this score is if half the N students had a score of 0, and the other half had a score of 2280:

[tex]mean=\frac{\frac{N}{2}\cdot0+\frac{N}{2}\cdot2280}{N}=\frac{N\cdot1140}{N}=1140[/tex]

From this example, none of the students had a score of exactly 1140, and half of them had a score less than 1140. So, we can conclude that the first three statements must not be true.

Step 2

Analyze the other statements.

The fourth statement must not be true because we can't conclude anything for sure for next year's scores based on the last year's scores.

Let's analyze the fifth statement. Suppose it must not be true, i.e., all the freshman students had scores equal to or less than 900. Then, since the mean score can't be greater than the maximum score, the mean score would be no more than 900. Wich is false because it was 1140 > 900.

Therefore, the fifth statement must be true.

Answer

The only claim that must be true is:

Last year, at least one of their freshman students had a score of more than 900 on the exam.

If a ^20 = (a^n)^m, which of the following could be values for m and n?obA) m = -5, n = -4B) m = 10, n = 10C) m = 22, n = -2D) m = 15, n = 5d

Answers

a ^20 = (a^n)^m

When we have a number raised to a power two time, we can multiply the powers;

(a^n)^m = a ^ (n x m)

So, since both sides have the same base:

a^20 = a^ (nxm)

20 = n x m

So, the product of n and m must be 20

A) -5 x -4 = 20

B) 10 x 10 =100

c) 22 x -2 =-44

d)15 x 5 = 75

The correct answer is A.

if 5 plus 5 is 10 and 44 plus 87 plus 98 plus 1415 is what???

Answers

Answer:

5+5=10

44+87= 131

98+131=229

1415+229=1644

Step-by-step explanation:

the answer is 1644 so all you need to kno w is to follow the procedure you use for the 5 plus 5 method

Shay created the table below to graph the equation r=2-sin theta (rounded to the hundredths place). Analyze the table. Did Shay make a mistake? If there is a mistake, which point is incorrect

Answers

The given equation is

[tex]r=2-sin\theta[/tex]

To find the incorrect point, we will substitute the value of theta in each point and find the value of r to the nearest 2 decimal points and compare it with the value of r of the point

[tex]\begin{gathered} \theta=0 \\ r=2-sin0 \\ r=2-0 \\ r=2 \end{gathered}[/tex]

Answer A is correct

[tex]\begin{gathered} \theta=\frac{\pi}{6} \\ r=2-sin\frac{\pi}{6} \\ r=2-\frac{1}{2} \\ r=1.5 \end{gathered}[/tex]

Answer B is correct

[tex]\begin{gathered} \theta=\frac{\pi}{3} \\ r=2-sin\frac{\pi}{3} \\ r=1.13 \end{gathered}[/tex]

Since the value of corresponding r is 2.09, then

Answer C is incorrect

The answer is C

???. Can you help me .???I have to find the simple interest earned to the nearest cent for each principle, interest rate, and time

Answers

Given:

Principal amount, P = $640

Time, T = 2 years

Interest rate, R = 3%

Let's find the simple interest.

To find the simple interest, apply the Simple Interest formula:

[tex]I=\frac{P\ast R\ast T}{100}[/tex]

Substitute values into the formula:

[tex]\begin{gathered} I=\frac{640\ast3\ast2}{100} \\ \\ I=\frac{3840}{100} \\ \\ I=38.40 \end{gathered}[/tex]

Therefore, the simple interest to the nearest cent is $38.40

ANSWER:

$38.40

Please help me with the equation part of this question thank you

Answers

[tex]d=60t[/tex]

B) As requested, let's focus on figuring out the equation. Note from the table that this is a proportional relationship, the more time goes by the more the speed increases.

2) Proportional relationships are linear equations, with the y-intercept passing through the origin. So, we can write out this equation simply:

[tex]d=60t[/tex]

Note: the y-intercept is equal to zero.

Thus, this is the answer.

The diameter of a circle is 20 kilometers. What is the angle measure of an arc bounding a sector with area 10pi square kilometers?Give the exact answer in simplest form. ____°. (pi, fraction,)

Answers

The area of a circular sector is given by:

[tex]A=\frac{1}{4}\cdot\pi\cdot d^2\cdot\frac{\theta}{360}[/tex]

Where:

π ≈ 3.14159

d = diameter of the circle

θ = angle of the circular sector

In our problem we have that:

[tex]\begin{gathered} A=10\cdot\pi\cdot km^2 \\ d=20\operatorname{km} \end{gathered}[/tex]

And we need to find the value of the angle θ. So in order to solve the problem, we replace the given data in the formula of above:

[tex]\begin{gathered} A=\frac{1}{4}\cdot\pi\cdot d^2\cdot\frac{\theta}{360^{\circ}} \\ 10\cdot\pi\cdot km^2=\frac{1}{4}\cdot\pi\cdot(20\operatorname{km})^2\cdot\frac{\theta}{360^{\circ}} \end{gathered}[/tex]

And now we solve for θ:

[tex]\begin{gathered} 10\cdot\pi\cdot km^2=\frac{1}{4}\cdot\pi\cdot400\cdot km^2\cdot\frac{\theta}{360^{\circ}} \\ 10=100\cdot\frac{\theta}{360^{\circ}} \\ 360^{\circ}\cdot\frac{10}{100}=\theta \\ \theta=36^{\circ} \end{gathered}[/tex]

So the answer is that the angle of the circular sector is: 36°

5. Math home work thanks type the answer out domain and range

Answers

Answer:

Explanation:

Given the below quadratic function in vertex form;

[tex]g(x)=-0.25(x-1)^2+19[/tex]

A quadratic equation in vertex form is generally given as;

[tex]y=a(x-h)^2+k[/tex]

where (h, k) is the coordinate of the vertex.

When a

U is defined as the set of all integers. Consider the following sets:A = {1, 2, 3, 4, 5}B = {x| 0 < x < 5}C = {p|P is an even prime number}D = {4. 5. 6. 7}E = {x| x is a square number less than 50}Find BDGroup of answer choices40, 1, 2, 3, 4, and 54 and 50, 1, 2, 3, 4, 5, 6, and 7

Answers

We will have te following

BUD:

[tex]B\cup D\colon1,2,3,4,5,6,7[/tex]

So BUD is 1,2,3,4,5,6 & 7.

Find the angle of taper on the steel bar shown if it is equal to twice.

Answers

The angle m can be found by noticing that:

cos m = 22.5/25

That's so because if we divide the triangle formed by the taper into two others, we get two right triangles.

Each right triangle has the angle m, the adjacent leg measuring 22.5 mm, and the hypotenuse measuring 25.0 mm.

So, using the formula for the cosine, we get the above relation. Then, solving this equation, we have:

cos m = 22.5/25

m = arccos 22.5/25

m ≅ 25.84º

Therefore, the angle of the taper is:

2 * m = 2 * 25.84º = 51.68º

Now, in order to convert this result using arc minutes, we need to remember that each 1º corresponds to 60' (60 arc minutes). Thus, we have:

1º --- 60'

0.68º --- x

So, cross multiplying those values, we find:

1º * x = 0.68º * 60'

x = (0.68º/1º) * 60'

x = 0.68 * 60'

x = 40.8'

x ≅ 41'

Therefore, the answer is 51º41'.

in the triangle abc a =65 b =58 identity the longest side of the triangle

Answers

We know two angles of a triangle, ∠A = 65° and ∠B = 58°, and we have to identify the longest side.

The longest side will be the one that is opposite to the widest angle. In our case, we don't know the measure of C, but we know that the sum of the three measures has to be 180°, so we can calculate it as:

[tex]\begin{gathered} m\angle A+m\angle B+m\angle C=180\degree \\ 65+58+m\angle C=180 \\ m\angle C=180-65-58 \\ m\angle C=57\degree \end{gathered}[/tex]

As the widest angle is at vertex A, the longest side will be its opposite, which correspond to the side formed by the other two vertices: B and C.

The longest side is BC.

Which equation can Pablo use to find p the regular price of the shirt

Answers

The final price of the shirt is given by the regular price minus the discount value. Since the final price is $28, the regular price is p, and the discount is $16, the equation is

[tex]p-16=28[/tex]

If we add 16 to both sides of the equation, we have

[tex]\begin{gathered} p-16+16=28+16 \\ p=28+16 \end{gathered}[/tex]

If we invert the order of the equality, we get the last option as the answer

[tex]16+28=p[/tex]

Circumference and the area of a circle with radius 5 ft you

Answers

The circunference formula is given by

[tex]C=2\pi r[/tex]

where r is the radius. Since r measures 5 ft, we have

[tex]\begin{gathered} C=2\pi\cdot5 \\ C=10\pi \end{gathered}[/tex]

By taking into account that Pi is 3.14, the circuference is equal to 31.4 ft.

On the other hand, the area formula is given by

[tex]A=\pi r^2[/tex]

Then, by substituting r=5 into this formula, we get

[tex]\begin{gathered} A=(3.14)(5^2) \\ A=3.14\times25 \\ A=78.5ft^2 \end{gathered}[/tex]

then, the area is equal to 78.5 square feet

Use point-slope form to write the equation of a line that passes through the point (-8,-16)(−8,−16) with slope 11.

Answers

The general point-slope equation of a line is:

[tex]y=m\cdot(x-x_0)+y_0\text{.}[/tex]

Where:

• m is the slope of the line,

,

• and (x0,y0) are the coordinates of one of the points of the line.

In this problem we have:

• m = 11,

,

• (x0,y0) = (-8,-16).

Replacing these values in the general equation, we have:

[tex]y=11\cdot(x+8)-16[/tex]

Answer

The point-slope equation of the line is:

[tex]y=11\cdot(x+8)-16[/tex]

Other Questions
A recording company obtains the blank cds used to produce its labels from three compact disk manufacturers: i, ii, and iii. The quality control department of the company has determined that 3% of the compact disks produced by manufacturer i are defective, 4% of those produced by manufacturer ii are defective, and 2% of those produced by manufacturer iii are defective. Manufacturers i, ii, and iii supply 36%, 52%, and 12%, respectively, of the compact disks used by the company. What is the probability that a randomly selected label produced by the company will contain a defective compact disk?. 4. A 1.5 m tall man is standing 2m away from the concave lens (remember f is negative) of a peephole with a focal length of 3.0 cm. a) What is the distance to the image in centimeters? b) What is the magnification of the image in meters? How to cope up with anxiety at school when u r helpless. Question Joan invested $1,420 at the start of the year and found she had $1,621.40 at the end of the year. What is the annual effective yield of her investment? Input your answer as a percentage rounded to two decimal. what made the trans-Saharan trade route possible it's a graph, I need help with the first one to understand how to do the rest. Please draw it clearly and understandably. From the perspective of a corrupt government, disappearances provide which of the following benefits?O No reason need be provided for the arrestO The government does not have to provide an attorney or a trialFear spreads throughout the society, often promoting complianceAll of the above Find the equation (in terms of x) of the line through the points (-4,-5) and (1,5) Translate the figure 1 unit left and 3 units down.Plot all of the points of the translated figure.You may click a plotted point to delete it.-10--54 - -3 4 5 6 7 A right triangle has a hypotenuse of 18 feet and a side length opposite of 12 feet . What is the measure of angle A the nearest degree ? Exercise 2: Subject and verb agreement: Circle the correct form of the verb "to be" in parenth 1. Six miles (is, are) a long way to walk to school. 2. Louisa (has, have) two jobs. 3. Giovanni and Maria (has, have) a small apartment and three dogs. 4. The group (is, are) very large. 5. The jacket and the gloves (is, are) on the table. 6. Mr. Izaguerra (arrive, arrives) at work early every day. 7. The beer (is, are ) very good. 8. Nine hundred dollars (is, are) a very high rent for that old building. 9. Mr. Iwasaki (dance, dances) very well. 10. Fifty percent of the book (is, are ) about the Vice-President. Stress is typically ____________ for adolescent girls compared to adolescent boys. Find the ordered pairs for the x- and y-intercepts of the equation 8x - 2y = 16 and select the appropriate option below. The x-intercept is (-2, 0), the y-intercept is (0, 8). The x-intercept is (0, 2), the y-intercept is (-8, 0). The x-intercept is (2, 0), the y-intercept is (0, -8). The x-intercept is (0, -2), the y-intercept is (8, 0). Kim places $1,250 into a bank account that pays 4.25% simple interest. if she does not add or remove any money from the account, how much interest will she earn after 60 months (anwers must be written as money-rounded to the hundred to the place) i need help please with m and b#1 and graph About how far does the flag have to travel to complete full rotation?1. 37.68 ft2. 75.36 ft3. 150.72 ft4. 452.16 ftPlease explain if possible! jodie has 2 1/2 cases of soda to split between 5 families. what fraction of a case does each family receive? 16) A bottle of high blood pressure medication contains 90 tablets. Each tablet contains 150 mg of the active ingredient. How many grams of the active ingredient are in the entire bottle? Hint: 1 gram = 1,000 mg 2. What is the equation of the line that passes through (5, 2) and isperpendicular to y =10x + 7?AC.y10x +yX+1021y = 10x - 4810B.-+52+52D. can you PLEASE help me